Solving DE using power series (w/initial conditions)

Click For Summary
The discussion focuses on solving the differential equation y'' = y' + y with initial conditions y(0) = 0 and y(1) = 1. A power series solution is proposed, leading to a recurrence relation involving Fibonacci numbers. Participants identify a potential misprint in the problem, suggesting that y'(0) = 1 might be the correct initial condition instead of y(1) = 1. By adjusting the initial conditions to y(0) = 0 and y'(0) = 1, the series solution aligns with the expected outcome. This adjustment successfully resolves the problem, confirming the importance of accurate initial conditions in power series solutions.
Fuzedmind
Messages
9
Reaction score
0

Homework Statement



Solve the initial value problem y'' = y' + y where y(0) = 0 and y(1) = 1

derive the power series solution y(x) = <br /> \ \ \sum_{n=1}^{\infty}{(F_{n}x^n)/n!} \ \ where {Fn} is the sequence 0,1,1,2,3,5,8,13... of Fibonacci numbers defined by F0 = 0 and F1 = 1

Homework Equations


The Attempt at a Solution



I plugged in the series and got this equation:

<br /> \ \ \sum_{n=2}^{\infty}{(n)(n-1)c_{n}x^(n-2)} \ \ + \ \ \sum_{n=1}^{\infty}{c_{n}x^(n-1)} \ \ + \ \ \sum_{n=0}^{\infty}{c_{n}x^n} \ \ = x<br />

from which i got the recurrence equation:

Fn = Fn-1/n + Fn-2/n(n-1)

Problem is I don't know how to get F0 and F1 from the initial conditions. My teacher never taught us how to solve initial condition power series ODE's nor are there any examples in the book. Can anyone explain it to me?
 
Physics news on Phys.org
You are assuming y = c0 + c1x + c2x2+...

y(0) is just c0. And y'(0) would be c1 , which would be the usual conditions for an IVP. Are you sure you have the second boundary condition written correctly? My guess it is supposed to be y'(0) = 1.
 
no y(0) = 0 and y(1) = 1, I am looking at it right now
 
I don't have time to work through the solution right now, but what you have then is a boundary value problem, not an initial value problem as claimed. I'm suspecting a misprint. You might just try using y'(0)=1 and working it through to see if you get the correct answer.
 
I had a little time later in the day. Your text indeed has a misprint. If you take your equation with the initial conditions y(0) = 0 and y'(0) = 1, which give you the values of c0 and c1 and work out the series solution, you get exactly what the problem claims you will get.
 
Ok thanks a bunch man. I did that and it worked out for me too.
 
Question: A clock's minute hand has length 4 and its hour hand has length 3. What is the distance between the tips at the moment when it is increasing most rapidly?(Putnam Exam Question) Answer: Making assumption that both the hands moves at constant angular velocities, the answer is ## \sqrt{7} .## But don't you think this assumption is somewhat doubtful and wrong?

Similar threads

  • · Replies 2 ·
Replies
2
Views
2K
Replies
8
Views
3K
  • · Replies 3 ·
Replies
3
Views
2K
  • · Replies 7 ·
Replies
7
Views
2K
  • · Replies 1 ·
Replies
1
Views
2K
  • · Replies 11 ·
Replies
11
Views
3K
  • · Replies 9 ·
Replies
9
Views
3K
  • · Replies 4 ·
Replies
4
Views
2K
  • · Replies 2 ·
Replies
2
Views
1K
  • · Replies 14 ·
Replies
14
Views
2K